2021AMC10A - 数学 (10)

版权:美国数学协会(Mathematical Association of America, MAA)

第1题

2018 AMC 10B 试题 第1题

问题 计算 \(\frac{{\left( 2112 - 2021 \right)}^{2}}{169}\) 的值。

(A) 7 (B) 21 (C) 49 (D) 64 (E) 91

第1题解答

解答步骤

1. 理解题意

题目要求计算分式 \(\frac{{\left( 2112 - 2021 \right)}^{2}}{169}\) 的值。 我们首先需要计算括号内的差值,然后平方,最后除以分母 169。

2. 解题过程

步骤 1:计算括号内的差值 \[ 2112 - 2021 = 91 \] 所以原式变为: \[ \frac{91^2}{169} \]

步骤 2:分解分子和分母 观察 \(91\) 和 \(169\) 的关系: \[ 91 = 7 \times 13 \] \[ 169 = 13^2 \] 代入原式: \[ \frac{(7 \times 13)^2}{13^2} \]

步骤 3:化简分式 利用指数运算法则 \((a \cdot b)^2 = a^2 \cdot b^2\) : \[ \frac{7^2 \times 13^2}{13^2} \] 约去分子和分母的公因子 \(13^2\) : \[ 7^2 = 49 \]

3. 最终答案

最终答案是 \(49\) ,对应选项 (C)。
第2题

2018 AMC 10B 试题 第2题

门卡拉有一张 \( 4 \times 6 \) 的索引卡。如果她将其中一条边的长度缩短 1 英寸,这张卡的面积将变为 18 平方英寸。那么,如果她改为将另一条边的长度缩短 1 英寸,这张卡的面积将是多少平方英寸?

第2题解答

解答步骤

1. 理解题意

已知索引卡原始尺寸为 \(4 \times 6\) (单位:英寸)。 若将其中一条边缩短 1 英寸,面积变为 18 平方英寸。 我们需要求:若将另一条边缩短 1 英寸,面积是多少平方英寸。


2. 解题过程

设原始卡片的长为 \(l\) 、宽为 \(w\) ,且 \(l \times w = 4 \times 6 = 24\) 平方英寸。

情况 1:将一条边缩短 1 英寸后面积为 18

因此,第一次缩短的是长度为 4 的边。


情况 2:改为缩短另一条边

另一条边长度为 6,缩短 1 英寸后变为 5 英寸。 此时卡片尺寸为 \(4 \times 5\) 。

面积计算: \[ 4 \times 5 = 20 \]


3. 最终答案

最终面积是 \(20\) 平方英寸。
第3题

2018 AMC 10B 试题 第3题

假设黏土球在装入立方体前可以被重塑但不可压缩,半径为 \(2\) 的黏土球最多能完全装入边长为 \(6\) 的立方体中多少个?

(A) 3 (B) 4 (C) 5 (D) 6 (E) 7

第3题解答

解答步骤

1. 理解题意

题目给出一个半径为 \(2\) 的黏土球,在装入边长为 \(6\) 的立方体前可以被重塑(改变形状)但不可压缩(体积不变)。要求计算最多能完全装入多少个这样的黏土球。

关键点:

2. 解题过程

第一步:计算单个黏土球的体积

球的体积公式为 \(V_{\text{球}} = \frac{4}{3}\pi r^3\)

代入半径 \(r = 2\) : \[ V_{\text{球}} = \frac{4}{3}\pi \cdot 2^3 = \frac{4}{3}\pi \cdot 8 = \frac{32}{3}\pi \]

第二步:计算立方体的体积

立方体体积公式为 \(V_{\text{立方体}} = a^3\)

代入边长 \(a = 6\) : \[ V_{\text{立方体}} = 6^3 = 216 \]

第三步:计算理论最大球数

理论上最多能容纳的球数为: \[ N_{\text{理论}} = \frac{V_{\text{立方体}}}{V_{\text{球}}} = \frac{216}{\frac{32}{3}\pi} = \frac{216 \cdot 3}{32\pi} = \frac{648}{32\pi} = \frac{81}{4\pi} \]

第四步:估算实际最大整数球数

需要估算 \(\frac{81}{4\pi}\) 的值范围:

已知 \(\pi \approx 3.1416\) ,所以: \[ 4\pi \approx 12.5664 \]

计算上下界:

因此: \[ 6.23 < \frac{81}{4\pi} < 6.75 \]

由于球数必须是整数,且 \(\frac{81}{4\pi} < 7\) ,所以最大可能的整数球数为 \(6\) 。

3. 最终答案

最多能完全装入 6 个黏土球,对应选项 (D)
第4题

(A) \ \( 2\\frac{3}{4} \\) (B) \ \( 3\\frac{3}{4} \\) (C) \ \( 4\\frac{1}{2} \\) (D) \ \( 5\\frac{1}{2} \\) (E) \ \( 6\\frac{3}{4} \\)

第4题解答

解答步骤

1. 理解题意

题目要求比较两条路线的行驶时间差,并将时间差转换为分钟。 路线 A 全长 6 英里,全程平均速度为 30 英里/小时。 路线 B 全长 5 英里,但其中有一段 \( \frac{1}{2} \) 英里的学校区域,平均速度为 20 英里/小时;其余路段平均速度为 40 英里/小时。 我们需要计算路线 B 比路线 A 快多少分钟。


2. 计算路线 A 所需时间

路线 A 的长度 \( L_A = 6 \) 英里,速度 \( v_A = 30 \) 英里/小时。 行驶时间为: \[ t_A = \frac{L_A}{v_A} = \frac{6}{30} = \frac{1}{5} \ \text{小时} \]


3. 计算路线 B 所需时间

路线 B 全长 \( L_B = 5 \) 英里,其中:

学校区域行驶时间: \[ t_s = \frac{L_s}{v_s} = \frac{\frac{1}{2}}{20} = \frac{1}{40} \ \text{小时} \]

其余路段行驶时间: \[ t_r = \frac{L_r}{v_r} = \frac{\frac{9}{2}}{40} = \frac{9}{80} \ \text{小时} \]

路线 B 总时间: \[ t_B = t_r + t_s = \frac{9}{80} + \frac{1}{40} \] 通分(分母为 80): \[ t_B = \frac{9}{80} + \frac{2}{80} = \frac{11}{80} \ \text{小时} \]


4. 计算时间差(小时)

路线 B 比路线 A 快的时间为: \[ \Delta t = t_A - t_B = \frac{1}{5} - \frac{11}{80} \] 通分(分母为 80): \[ \Delta t = \frac{16}{80} - \frac{11}{80} = \frac{5}{80} = \frac{1}{16} \ \text{小时} \]


5. 将时间差转换为分钟

\[ \frac{1}{16} \ \text{小时} \times 60 \ \text{分钟/小时} = \frac{60}{16} = \frac{15}{4} = 3\frac{3}{4} \ \text{分钟} \]


6. 最终答案

路线 B 比路线 A 快 \( 3\frac{3}{4} \) 分钟,对应选项 (B)
第5题

2018 AMC 10B 试题 第5题

问题 六位数 \(\underline{2}\underline{0}\underline{2}\underline{1}\underline{0}\underline{\mathrm{A}}\) 仅当数字 \(\mathrm{A}\) 取某个特定值时是质数。请问 \(\mathrm{A}\) 的值是多少?

(A) 1 (B) 3 (C) 5 (D) 7 (E) 9

第5题解答

解答步骤

1. 理解题意

题目给出六位数 \(\underline{2}\underline{0}\underline{2}\underline{1}\underline{0}\underline{\mathrm{A}}\) ,其中 \(\mathrm{A}\) 是一个特定的个位数字,使得这个六位数是质数。我们需要从选项 (A) 1, (B) 3, (C) 5, (D) 7, (E) 9 中找出正确的 \(\mathrm{A}\) 值。

2. 解题过程

首先,我们列出所有可能的六位数形式: \[ 20210A \] 其中 \(A\) 是 0 到 9 的数字,但题目给出的选项是 1, 3, 5, 7, 9(均为奇数,因为若 \(A\) 为偶数,则整个数能被 2 整除,不是质数)。

我们逐一分析每个选项:

此时数为 \(202103\) 。 判断是否能被 11 整除:使用交错和(从左边开始,正负交替): \[ 2 - 0 + 2 - 1 + 0 - 3 = 0 \] 0 能被 11 整除,所以整个数能被 11 整除,不是质数。排除。

此时数为 \(202109\) 。 通过以上排除法,其余四个选项均导致整个数有大于 1 的因数,因此只有当 \(A = 9\) 时, \(202109\) 才可能是质数。 题目说明“仅当数字 \(\mathrm{A}\) 取某个特定值时是质数”,且已知答案对应 (E),因此 \(202109\) 是质数(可通过计算机代数软件验证)。

3. 最终答案

\(\mathrm{A}\) 的值是 9,对应选项 (E)。
第6题

题目

鸸鹋埃尔默在乡村道路上每两个相邻电线杆之间需要走44步等长的步伐。鸵鸟奥斯卡用12次等长的跳跃就能覆盖相同的距离。电线杆间距均匀,沿着这条路第41根电线杆与第一根电线杆正好相距1英里(5280英尺)。奥斯卡的跳跃比埃尔默的步伐长多少英尺?

(A) 8 (B) 9 (C) 10 (D) 11 (E) 15

第6题解答

解答步骤

1. 理解题意

2. 解题过程

步骤1:计算相邻电线杆之间的距离 从第1根电线杆到第41根电线杆,共有 \(41 - 1 = 40\) 个间隔。 每个间隔的距离为: \[ \frac{5280}{40} = 132 \text{ 英尺} \]

步骤2:计算埃尔默的步伐长度 埃尔默在两个相邻电线杆之间走44步,因此他的步伐长度为: \[ \frac{132}{44} = 3 \text{ 英尺} \]

步骤3:计算奥斯卡的跳跃长度 奥斯卡在两个相邻电线杆之间用12次跳跃覆盖相同距离,因此他的跳跃长度为: \[ \frac{132}{12} = 11 \text{ 英尺} \]

步骤4:计算跳跃与步伐的长度差 奥斯卡的跳跃比埃尔默的步伐长: \[ 11 - 3 = 8 \text{ 英尺} \]

3. 最终答案

最终答案是 \(8\) ,对应选项 (A)。
第7题

2018 AMC 10B 试题 第7题

(A) 170 (B) 174 (C) 其他选项未列出 (D) 170 (E) 174

第7题解答

解答步骤

1. 理解题意

题目给出正方形 \(ABCD\) ,点 \(E\) 与点 \(A\) 在直线 \(CD\) 所确定的相对半平面上,且 \(\angle CDE = 110^\circ\) 。点 \(F\) 在线段 \(AD\) 上,满足 \(DE = DF\) ,因此 \(\triangle DEF\) 是等腰三角形。要求计算 \(\angle AFE\) 的度数。


2. 解题过程

步骤 1:确定 \(\angle EDF\)

\[ \angle EDF = 360^\circ - \angle ADC - \angle CDE = 360^\circ - 90^\circ - 110^\circ = 160^\circ. \]


步骤 2:利用等腰三角形性质求 \(\angle DFE\)

\[ \angle DEF = \angle DFE. \] \[ \angle DEF + \angle DFE + \angle EDF = 180^\circ. \] 代入已知: \[ 2 \angle DFE + 160^\circ = 180^\circ \quad \Rightarrow \quad 2 \angle DFE = 20^\circ \quad \Rightarrow \quad \angle DFE = 10^\circ. \]


步骤 3:求 \(\angle AFE\)

\[ \angle AFE = 180^\circ - \angle DFE = 180^\circ - 10^\circ = 170^\circ. \]


3. 最终答案

\[ \boxed{170} \]

第8题

题目

答案(B):设一个两位可爱数为 \( \underline{a}\underline{b} \) ,其值为 \( 10a + b \) 。因此有 \( 10a + b = a + b^{2} \) ,即 \( 9a = b(b - 1) \) 。由此可得 9 整除 \( b(b - 1) \) 。若 \( 3 \mid b \) 且 \( 3 \mid (b - 1) \) ,则 \( 3 \mid 1 \) ,矛盾。因此要么 \( 9 \mid b \) ,要么 \( 9 \mid (b - 1) \) 。此外,由于 \( a \neq 0 \) ,可得 \( b > 1 \) 。因此 \( b \) 唯一可能的取值为 9。此时 \( 9a = 9 \cdot 8 \) ,即 \( a = 8 \) ,故唯一的两位可爱数是 89。

第8题解答

解答步骤

1. 理解题意

题目定义:一个两位正整数,如果等于其十位数字(非零)与个位数字平方的和,则称该数为“可爱数”。 设该两位数为 \(\underline{a}\underline{b}\) ,其中 \(a\) 为十位数字( \(a \in \{1, 2, \dots, 9\}\) ), \(b\) 为个位数字( \(b \in \{0, 1, \dots, 9\}\) )。 根据定义,该数满足: \[ 10a + b = a + b^2 \]


2. 解题过程

步骤 1:建立方程 由 \(10a + b = a + b^2\) ,移项得: \[ 10a - a = b^2 - b \] \[ 9a = b(b - 1) \]

步骤 2:分析整除性 由于 \(9a = b(b - 1)\) ,且 \(a\) 为正整数,因此 \(9\) 必须整除 \(b(b - 1)\) ,即: \[ 9 \mid b(b - 1) \]

步骤 3:排除矛盾情况 若 \(3 \mid b\) 且 \(3 \mid (b - 1)\) ,则 \(3 \mid 1\) ,矛盾。 因此, \(b\) 和 \(b - 1\) 不能同时被 3 整除,即 \(9\) 必须整除 \(b\) 或 \(b - 1\) 中的一个。

步骤 4:确定 \(b\) 的可能取值

结合 \(a \neq 0\) 及 \(9a = b(b - 1) > 0\) ,得 \(b > 1\) 。 因此, \(b\) 的可能取值为 \(b = 9\) 。

步骤 5:求解 \(a\) 代入 \(b = 9\) 到方程 \(9a = b(b - 1)\) : \[ 9a = 9 \times 8 = 72 \] \[ a = 8 \]

步骤 6:验证 两位数为 \(89\) ,计算: \[ 8 + 9^2 = 8 + 81 = 89 \] 满足条件。


3. 最终答案

满足条件的两位“可爱数”只有 89,因此个数为 1。 对应选项为 (B)

最终答案是 1,对应选项 (B)

第9题

(B) \( \frac{4}{9} \) (C) \( \frac{5}{9} \) (D) \( \frac{9}{16} \) (E) \( \frac{5}{8} \)

第9题解答

解答步骤

1. 理解题意

题目指出,投掷一个不公平的骰子时,出现偶数的概率是奇数的3倍。我们需要将此骰子投掷两次,并求两次投掷所得数字之和为偶数的概率。

设出现奇数的概率为 \( p \) ,则出现偶数的概率为 \( 3p \) 。由于所有可能结果的概率之和为1,我们有: \[ p + 3p = 1 \] 解得: \[ 4p = 1 \quad \Rightarrow \quad p = \frac{1}{4} \] 因此:

\[ \left( \frac{3}{4} \right) \times \left( \frac{3}{4} \right) = \frac{9}{16} \]

\[ \left( \frac{1}{4} \right) \times \left( \frac{1}{4} \right) = \frac{1}{16} \]

\[ \frac{9}{16} + \frac{1}{16} = \frac{10}{16} = \frac{5}{8} \]

3. 最终答案

最终答案是 \( \frac{5}{8} \) ,对应选项 (E)。
第10题

(A) 70 (B) 84 (C) 105 (D) 126 (E) 140

第10题解答

解答步骤

1. 理解题意

艾米丽沿河岸以恒定速度行走,船在河中沿直线航道匀速行驶,艾米丽的速度比船快。


3. 建立方程

情况 1:艾米丽从船尾走向船头(与船同向)

相对速度 = \( u - v = 1 - v \) 所用步数 = 210 ⇒ 所用时间 = \( 210 \) (因为每步时间 = 1 单位时间,速度 = 1 步/单位时间) 在这段时间内,艾米丽相对于船走了 \( L \) 步的长度:

\[ 210 \times (1 - v) = L \] \[ \text{(1)} \quad 210(1 - v) = L \]

情况 2:艾米丽从船头走向船尾(与船反向)

相对速度 = \( u + v = 1 + v \) 所用步数 = 42 ⇒ 所用时间 = \( 42 \) 在这段时间内,艾米丽相对于船走了 \( L \) 步的长度:

\[ 42 \times (1 + v) = L \] \[ \text{(2)} \quad 42(1 + v) = L \]


4. 联立方程求解

由 (1) 和 (2) 得: \[ 210(1 - v) = 42(1 + v) \] \[ 210 - 210v = 42 + 42v \] \[ 210 - 42 = 42v + 210v \] \[ 168 = 252v \] \[ v = \frac{168}{252} = \frac{2}{3} \]

代入 (2): \[ L = 42 \times \left( 1 + \frac{2}{3} \right) = 42 \times \frac{5}{3} = 14 \times 5 = 70 \]


5. 最终答案

船的长度为 \( 70 \) 步,对应选项 (A)
第11题

第11题解答

解答步骤

第12题

1. 理解题意

题目给出数 \( N \) 的九进制表示: \( 27{,}006{,}000{,}052_9 \) 。要求计算 \( N \) 除以 5 的余数。

2. 解题过程

首先将九进制数展开为十进制表达式: \[ N = 2 \cdot 9^{10} + 7 \cdot 9^{9} + 6 \cdot 9^{6} + 5 \cdot 9^{1} + 2 \cdot 9^{0} \]

我们需要计算 \( N \mod 5 \) 。由于 \( 9 \equiv -1 \pmod{5} \) ,所以:

将所有余数相加: \[ 2 + (-7) + 6 + (-5) + 2 = -2 \equiv 3 \pmod{5} \]

3. 最终答案

\( N \) 除以 5 的余数是 3,对应选项 (D)。


第13题

1. 理解题意

有 6 个球,每个球独立地以相等概率被涂成黑色或白色。需要计算每个球的颜色都与其余 5 个球中超过一半(即至少 3 个)的球颜色不同的概率。

2. 解题过程

设黑球为 B,白球为 W。

"每个球的颜色都与其余 5 个球中超过一半的球颜色不同"意味着:

- 如果颜色分布不是 3-3,比如 4-2、5-1 或 6-0,那么多数颜色的球会面对更多同色球,不满足条件。 - 例如:4黑2白时,每个黑球面对其余 5 个球(3黑2白),只有 2 个颜色不同(白球),不满足"超过一半"(需要至少 3 个不同)。

因此,唯一满足条件的情况是 3黑3白。

3. 最终答案

概率 = \(\frac{20}{64} = \frac{5}{16}\)

由于题目只要求推演步骤,且已知答案对应选项,这里给出计算过程。

第12题

题目

(A) \(\frac{1}{6}\) (B) \(\frac{1}{4}\) (C) \(\frac{5}{16}\) (D) \(\frac{1}{2}\)

第12题解答

解答步骤

1. 理解题意

题目要求将6个球随机且独立地涂成黑色或白色,每种颜色的概率相等。我们需要计算每个球的颜色都与超过半数的其他5个球不同的概率。 \[ \binom{6}{3} = \frac{6!}{3! \cdot 3!} = \frac{720}{6 \times 6} = 20 \] \[ \text{概率} = \frac{\text{有利情况数}}{\text{总情况数}} = \frac{20}{64} = \frac{5}{16} \]

3. 最终答案

最终答案是 \(\frac{5}{16}\) ,对应选项 (C)。
第13题

等腰三角形 \(ABC\) 中, \(AB = AC = 3\sqrt{6}\) ,且一个半径为 \(5\sqrt{2}\) 的圆与直线 \(AB\) 相切于点 \(B\) ,与直线 \(AC\) 相切于点 \(C\) 。求经过顶点 \(A\) 、 \(B\) 和 \(C\) 的圆的面积是多少?

(A) \(24\pi\) (B) \(25\pi\) (C) \(26\pi\) (D) \(27\pi\) (E) \(28\pi\)

第13题解答

解答步骤

1. 理解题意

等腰三角形 \(ABC\) 中, \(AB = AC = 3\sqrt{6}\) ,且存在一个半径为 \(5\sqrt{2}\) 的圆,与直线 \(AB\) 相切于点 \(B\) ,与直线 \(AC\) 相切于点 \(C\) 。我们需要求经过顶点 \(A\) 、 \(B\) 和 \(C\) 的圆的面积。

2. 解题过程

设半径为 \(5\sqrt{2}\) 的圆的圆心为 \(O\) 。由于该圆与直线 \(AB\) 相切于点 \(B\) ,与直线 \(AC\) 相切于点 \(C\) ,根据切线的性质,半径与切线垂直,因此: \[ \angle ABO = 90^\circ, \quad \angle ACO = 90^\circ. \]

考虑四边形 \(ABOC\) ,其中 \(\angle ABO\) 和 \(\angle ACO\) 均为直角,因此: \[ \angle ABO + \angle ACO = 180^\circ. \] 这说明四边形 \(ABOC\) 的对角互补,因此四边形 \(ABOC\) 是圆内接四边形。

由于四边形 \(ABOC\) 内接于圆,且 \(\angle ABO = 90^\circ\) ,根据圆内接四边形的性质,对角互补,可得 \(\angle AOC = 90^\circ\) 。进一步,由对称性可知, \(AO\) 是经过点 \(A\) 、 \(B\) 、 \(C\) 的圆的直径。

接下来,计算 \(AO\) 的长度。在直角三角形 \(ABO\) 中,已知: \[ AB = 3\sqrt{6}, \quad OB = 5\sqrt{2}. \] 应用勾股定理: \[ AO = \sqrt{AB^2 + OB^2} = \sqrt{(3\sqrt{6})^2 + (5\sqrt{2})^2}. \] 计算各项: \[ (3\sqrt{6})^2 = 9 \times 6 = 54, \quad (5\sqrt{2})^2 = 25 \times 2 = 50. \] 因此: \[ AO = \sqrt{54 + 50} = \sqrt{104} = 2\sqrt{26}. \] 由于 \(AO\) 是经过点 \(A\) 、 \(B\) 、 \(C\) 的圆的直径,该圆的半径为: \[ R = \frac{AO}{2} = \frac{2\sqrt{26}}{2} = \sqrt{26}. \]

3. 最终答案

经过顶点 \(A\) 、 \(B\) 、 \(C\) 的圆的面积为: \[ \pi R^2 = \pi (\sqrt{26})^2 = 26\pi. \] 因此,正确答案为选项 (C) \(26\pi\) .
第14题

(A) \(y\) 轴 (B) 直线 \(x = 1\) (C) 原点 (D) 点 \(\left( {\frac{1}{2},0}\right)\) (E) 点 \((1,0)\)

第14题解答

解答步骤

1. 理解题意

函数定义为: \[ f(x) = \left| \lfloor x \rfloor \right| - \left| \lfloor 1 - x \rfloor \right| \] 其中 \(\lfloor x \rfloor\) 表示不超过 \(x\) 的最大整数。我们需要判断该函数图像关于哪一项对称。选项包括: 此时 \(\lfloor x \rfloor = n\) , \(\lfloor 1 - x \rfloor = \lfloor 1 - n \rfloor\) 。 计算 \(f(x)\) : \[ f(x) = |n| - |1 - n| \] - 若 \(n \geq 1\) ,则 \(|n| = n\) , \(|1 - n| = n - 1\) ,故: \[ f(x) = n - (n - 1) = 1 \] - 若 \(n \leq 0\) ,则 \(|n| = -n\) , \(|1 - n| = 1 - n\) ,故: \[ f(x) = -n - (1 - n) = -1 \]

此时 \(\lfloor x \rfloor = n\) , \(\lfloor 1 - x \rfloor = \lfloor 1 - n - r \rfloor\) 。 由于 \(0 < r < 1\) ,有 \(0 < 1 - r < 1\) ,故: \[ \lfloor 1 - n - r \rfloor = -n \] 因此: \[ f(x) = |n| - |-n| = |n| - |n| = 0 \]

步骤 3:检验对称性条件 对称关于点 \(\left( \frac{1}{2}, 0 \right)\) 的条件是: \[ f(1 - x) = -f(x) \] 计算 \(f(1 - x)\) : \[ f(1 - x) = \left| \lfloor 1 - x \rfloor \right| - \left| \lfloor 1 - (1 - x) \rfloor \right| = \left| \lfloor 1 - x \rfloor \right| - \left| \lfloor x \rfloor \right| = -f(x) \] 因此,对任意 \(x\) ,有: \[ f(1 - x) = -f(x) \] 这说明点 \((x, f(x))\) 和点 \((1 - x, -f(x))\) 关于点 \(\left( \frac{1}{2}, 0 \right)\) 对称,因为该点是这两点的中点。

步骤 4:排除其他选项 通过具体值验证其他对称性不成立:

取 \(t = 0\) ,则 \(f(1) = 1\) , \(f(1) = 1 \neq -1\) ,不成立。


3. 最终答案

函数图像关于点 \(\left( \frac{1}{2}, 0 \right)\) 对称,对应选项 (D)。
第15题

(A) 9 (B) 10 (C) 11 (D) 13 (E) 17

第15题解答

解答步骤

1. 理解题意

由于平板不平行于地面,支柱顶端共面,因此平板可视为一个平面。设六边形边长为 \(s\) ,建立坐标系,利用共面条件求解。


2. 建立坐标系与几何关系

将正六边形置于平面 \(z=0\) ,中心在原点,顶点按逆时针排列: \[ A = (s, 0, 0), \quad B = \left(\frac{s}{2}, \frac{\sqrt{3}}{2}s, 0\right), \quad C = \left(-\frac{s}{2}, \frac{\sqrt{3}}{2}s, 0\right), \] \[ D = (-s, 0, 0), \quad E = \left(-\frac{s}{2}, -\frac{\sqrt{3}}{2}s, 0\right), \quad F = \left(\frac{s}{2}, -\frac{\sqrt{3}}{2}s, 0\right). \]

已知支柱高度: \(z_A = 12\) , \(z_B = 9\) , \(z_C = 10\) 。

设平板平面方程为: \[ z = px + qy + r. \] 代入 \(A, B, C\) 坐标:


3. 解平面方程参数

由 (1) 得 \(r = 12 - ps\) 。

(2) 减 (1): \[ 9 - 12 = p\frac{s}{2} - ps + q\frac{\sqrt{3}}{2}s \] \[ -3 = -\frac{p s}{2} + q\frac{\sqrt{3}}{2}s \] \[ -3 = s\left(-\frac{p}{2} + \frac{\sqrt{3}}{2}q\right) \quad (4) \]

(3) 减 (1): \[ 10 - 12 = -p\frac{s}{2} - ps + q\frac{\sqrt{3}}{2}s \] \[ -2 = -\frac{3ps}{2} + q\frac{\sqrt{3}}{2}s \] \[ -2 = s\left(-\frac{3p}{2} + \frac{\sqrt{3}}{2}q\right) \quad (5) \]

(5) 减 (4): \[ -2 - (-3) = s\left[-\frac{3p}{2} + \frac{\sqrt{3}}{2}q + \frac{p}{2} - \frac{\sqrt{3}}{2}q\right] \] \[ 1 = s(-p) \] \[ p = -\frac{1}{s}. \]

代入 (4): \[ -3 = s\left[-\frac{1}{2}\cdot\left(-\frac{1}{s}\right) + \frac{\sqrt{3}}{2}q\right] \] \[ -3 = s\left[\frac{1}{2s} + \frac{\sqrt{3}}{2}q\right] \] \[ -3 = \frac{1}{2} + \frac{\sqrt{3}}{2} s q \] \[ -\frac{7}{2} = \frac{\sqrt{3}}{2} s q \] \[ q = -\frac{7}{\sqrt{3}\, s}. \]

由 (1): \[ r = 12 - \left(-\frac{1}{s}\right) s = 12 + 1 = 13. \]

因此平面方程: \[ z = -\frac{1}{s}x - \frac{7}{\sqrt{3}\, s}y + 13. \]


4. 求 \(E\) 处高度

\(E = \left(-\frac{s}{2}, -\frac{\sqrt{3}}{2}s, 0\right)\) ,代入平面方程: \[ z_E = -\frac{1}{s}\cdot\left(-\frac{s}{2}\right) - \frac{7}{\sqrt{3}\, s} \cdot \left(-\frac{\sqrt{3}}{2}s\right) + 13 \] \[ z_E = \frac{1}{2} + \frac{7}{2} + 13 \] \[ z_E = 4 + 13 = 17. \]


5. 最终答案

位于 \(E\) 处的支柱高度是 \(\mathbf{17}\) 米,对应选项 (E)

\[ \boxed{17} \]

第16题

2018 AMC 10B 试题 第18题

如图,某农场主将一块矩形田地划分为 \(2 \times 2\) 的网格,共4个矩形区域。每个区域将种植一种作物:玉米、小麦、大豆或马铃薯。农场主不希望任何相邻区域(共享边界)种植玉米和小麦,也不希望任何相邻区域种植大豆和马铃薯。在这些限制条件下,农场主有多少种不同的作物种植方案?

(A) 12 (B) 64 (C) 84 (D) 90 (E) 144

第16题解答

解答步骤

1. 理解题意

农场主有一块 \(2 \times 2\) 的矩形田地,划分为 4 个区域,按顺时针方向依次编号为 1(左上)、2(右上)、3(右下)、4(左下)。每个区域种植一种作物:玉米(C)、小麦(W)、大豆(S)或马铃薯(P)。限制条件为:

步骤 3:计算总数

\[ 36 + 16 + 32 = 84 \]


3. 最终答案

满足条件的种植方案总数为 \(\boxed{84}\) ,对应选项 (C)。

第17题

第19题

一个半径为1的圆盘在边长为 \( s > 4 \) 的正方形内部紧贴边界滚动一周,扫过的区域面积为 \( A \) 。另一个半径为1的圆盘在同一个正方形外部紧贴边界滚动一周,扫过的区域面积为 \( 2A \) 。若 \( s \) 可表示为 \( a + \frac{b\pi }{c} \) 的形式,其中 \( a,b,c \) 均为正整数且 \( b \) 与 \( c \) 互质,求 \( a + b + c \) 的值?

(A) 10 (B) 11 (C) 12 (D) 13 (E) 14

第17题解答

解答步骤

1. 理解题意

半径为 1 的圆盘在正方形内部紧贴边界滚动一周,扫过的区域面积为 \(A\) ;另一个半径为 1 的圆盘在正方形外部紧贴边界滚动一周,扫过的区域面积为 \(2A\) 。已知正方形边长 \(s > 4\) ,要求根据面积关系求出 \(s\) 的表达式 \(a + \frac{b\pi}{c}\) ,并计算 \(a + b + c\) 的值。


2. 内部滚动扫过的区域面积 \(A\)

内部滚动的圆盘中心轨迹是一个正方形,其边长为 \(s - 2\) (因为圆盘半径 1,中心到边的距离为 1,所以中心轨迹正方形比原正方形边长少 2)。但扫过的区域并不是简单的环形,而是原正方形去掉中心一个边长为 \(s - 4\) 的正方形(因为圆盘贴着内部滚动时,中心离边界 1,圆盘边缘离边界 0,所以中心轨迹内侧距离原边界 2,因此未扫到的中心区域边长是 \(s - 4\) ),并且四个角上未扫到的区域是四个 \(1 \times 1\) 正方形去掉四分之一圆(半径 1)。

更直接的方法(根据答案提示):

因此: \[ \text{未扫到的总面积} = (s - 4)^2 + (4 - \pi) \] \[ A = s^2 - \left[(s - 4)^2 + (4 - \pi)\right] \] \[ A = s^2 - (s^2 - 8s + 16 + 4 - \pi) \] \[ A = s^2 - s^2 + 8s - 20 + \pi \] \[ A = 8s - 20 + \pi \]


3. 外部滚动扫过的区域面积 \(2A\)

外部滚动的圆盘中心轨迹是边长为 \(s + 2\) 的正方形(中心到原正方形边的距离为 1,所以轨迹边长 \(s + 2\) )。扫过的区域是四个矩形(每个长 \(s\) 、宽 2)加上四个四分之一圆(半径 2)。

四个矩形面积: \(4 \times (s \times 2) = 8s\) 四个四分之一圆(半径 2)面积: \(4 \times \frac{1}{4} \pi (2^2) = 4\pi\)

所以: \[ \text{外部滚动扫过的面积} = 8s + 4\pi \] 已知它等于 \(2A\) : \[ 8s + 4\pi = 2(8s - 20 + \pi) \]


4. 解方程求 \(s\)

\[ 8s + 4\pi = 16s - 40 + 2\pi \] \[ 4\pi - 2\pi + 40 = 16s - 8s \] \[ 2\pi + 40 = 8s \] \[ s = 5 + \frac{\pi}{4} \]

因此 \(a = 5, b = 1, c = 4\) ,且 \(b, c\) 互质。


5. 计算 \(a + b + c\)

\[ a + b + c = 5 + 1 + 4 = 10 \]


6. 最终答案

最终答案是 \(10\) ,对应选项 (A)。
第18题

(A) 4 (B) 6 (C) 8 (D) 12 (E) 16

第18题解答

解答步骤

1. 理解题意

题目要求找出所有正整数有序对 \((b, c)\) ,使得两个方程: \[ x^{2} + bx + c = 0 \quad \text{和} \quad x^{2} + cx + b = 0 \] 都没有两个不同的实数解。

一个一元二次方程没有两个不同的实数解,当且仅当其判别式小于或等于零。因此,我们需要: \[ b^{2} - 4c \leq 0 \quad \text{和} \quad c^{2} - 4b \leq 0。 \]

2. 解题过程

步骤 1:列出不等式条件

由判别式条件可得: \[ b^{2} \leq 4c \quad \text{和} \quad c^{2} \leq 4b。 \] 由于 \(b\) 和 \(c\) 是正整数,我们可以从这两个不等式推导出 \(b\) 和 \(c\) 的可能取值范围。

步骤 2:推导 \(b\) 的取值范围

将两个不等式相乘: \[ (b^{2})(c^{2}) \leq (4c)(4b) \quad \Rightarrow \quad b^{2}c^{2} \leq 16bc。 \] 由于 \(b > 0\) 且 \(c > 0\) ,两边同时除以 \(bc\) (正数)得: \[ bc \leq 16。 \] 另外,从 \(c^{2} \leq 4b\) 可得 \(c \leq 2\sqrt{b}\) ,但更直接的方法是利用 \(b^{2} \leq 4c\) 和 \(c^{2} \leq 4b\) 进行迭代检验。

由 \(b^{2} \leq 4c\) 和 \(c^{2} \leq 4b\) ,代入 \(c^{2} \leq 4b\) 到第一个不等式: \[ b^{2} \leq 4c \quad \Rightarrow \quad c \geq \frac{b^{2}}{4}。 \] 同时, \(c^{2} \leq 4b\) 意味着 \(c \leq 2\sqrt{b}\) 。因此: \[ \frac{b^{2}}{4} \leq c \leq 2\sqrt{b}。 \] 由于 \(c\) 是正整数,且 \(\frac{b^{2}}{4} \leq 2\sqrt{b}\) ,两边乘以 4: \[ b^{2} \leq 8\sqrt{b} \quad \Rightarrow \quad b^{4} \leq 64b \quad \Rightarrow \quad b^{3} \leq 64。 \] 所以: \[ b \leq 4。 \] 因此, \(b\) 的可能取值为 \(1, 2, 3, 4\) 。

步骤 3:对每个 \(b\) 值找出满足条件的 \(c\)

我们需要同时满足: \[ c \geq \frac{b^{2}}{4} \quad \text{和} \quad c^{2} \leq 4b。 \]

\[ c \geq \frac{1}{4} \quad \text{和} \quad c^{2} \leq 4 \Rightarrow c \leq 2。 \] 所以 \(c = 1, 2\) 。检查: - \((1,1)\) : \(1^{2} \leq 4 \cdot 1\) 成立, \(1^{2} \leq 4 \cdot 1\) 成立。 - \((1,2)\) : \(1^{2} \leq 4 \cdot 2\) 成立, \(2^{2} \leq 4 \cdot 1\) 成立( \(4 \leq 4\) )。 得到 2 个有序对。

\[ c \geq \frac{4}{4} = 1 \quad \text{和} \quad c^{2} \leq 8 \Rightarrow c \leq 2。 \] 所以 \(c = 1, 2\) 。检查: - \((2,1)\) : \(4 \leq 4 \cdot 1\) 成立( \(4 \leq 4\) ), \(1^{2} \leq 4 \cdot 2\) 成立。 - \((2,2)\) : \(4 \leq 8\) 成立, \(4 \leq 8\) 成立。 得到 2 个有序对。

\[ c \geq \frac{9}{4} = 2.25 \quad \text{和} \quad c^{2} \leq 12 \Rightarrow c \leq 3。 \] 所以 \(c = 3\) 。检查: - \((3,3)\) : \(9 \leq 12\) 成立, \(9 \leq 12\) 成立。 得到 1 个有序对。

\[ c \geq \frac{16}{4} = 4 \quad \text{和} \quad c^{2} \leq 16 \Rightarrow c \leq 4。 \] 所以 \(c = 4\) 。检查: - \((4,4)\) : \(16 \leq 16\) 成立, \(16 \leq 16\) 成立。 得到 1 个有序对。

步骤 4:统计所有有序对

所有满足条件的有序对为: \[ (1,1),\ (1,2),\ (2,1),\ (2,2),\ (3,3),\ (4,4)。 \] 共 6 个。

3. 最终答案

满足条件的正整数有序对共有 \(\boxed{6}\) 个,对应选项 (B)。
第19题

第21题

将20个球独立且随机地投入5个箱子中。设 \( p \) 为某个箱子最终有3个球、另一个箱子有5个球、其余三个箱子各有4个球的概率。设 \( q \) 为每个箱子最终都有4个球的概率。求 \( \frac{p}{q} \) 的值。

(A) 8 (B) 10 (C) 12 (D) 14 (E) 16

第19题解答

解答步骤

1. 理解题意

题目描述将 20 个球独立且随机地投入 5 个箱子中,每个球落入每个箱子的概率均为 \(\frac{1}{5}\) 。

要求计算 \(\frac{p}{q}\) 的值。


2. 解题思路

由于每个球独立且随机地落入 5 个箱子,总的分配方式数为 \(5^{20}\) ,但概率计算时分子分母都会出现 \(5^{20}\) ,因此 \(\frac{p}{q}\) 实际上等于两种分配方式数的比值。

我们使用多项式系数方法计算两种情况的分配方式数。


3. 计算 \(q\) 的分配方式数

每个箱子 4 个球,分配方式数为:

\[ N_q = \frac{20!}{4! \cdot 4! \cdot 4! \cdot 4! \cdot 4!} \]


4. 计算 \(p\) 的分配方式数

某个箱子有 3 个球,另一个箱子有 5 个球,其余三个箱子各有 4 个球。

分配方式数为:

\[ N_p = 20 \times \frac{20!}{3! \cdot 5! \cdot 4! \cdot 4! \cdot 4!} \]


5. 计算 \(\frac{p}{q}\)

由于概率 \(p = \frac{N_p}{5^{20}}\) , \(q = \frac{N_q}{5^{20}}\) ,所以:

\[ \frac{p}{q} = \frac{N_p}{N_q} \]

代入 \(N_p\) 和 \(N_q\) :

\[ \frac{p}{q} = \frac{20 \times \frac{20!}{3! \cdot 5! \cdot (4!)^3}}{\frac{20!}{(4!)^5}} \]

约去 \(20!\) :

\[ \frac{p}{q} = 20 \times \frac{(4!)^5}{3! \cdot 5! \cdot (4!)^3} \]

化简:

\[ \frac{p}{q} = 20 \times \frac{(4!)^2}{3! \cdot 5!} \]

计算阶乘:

\[ \frac{p}{q} = 20 \times \frac{576}{6 \times 120} = 20 \times \frac{576}{720} \]

\[ \frac{576}{720} = \frac{4}{5} \]

\[ \frac{p}{q} = 20 \times \frac{4}{5} = 16 \]


6. 最终答案

\[ \boxed{16} \]

对应选项 (E)。

第20题

2018 AMC 10B 试题 第22题

问题 在底面半径为5、高为12的直圆锥内部放置三个半径为 \( r \) 的全等球体。每个球体与其他两个球体相切,并且与圆锥的底面和侧面相切。求 \( r \) 的值。

(A) \( \frac{3}{2} \) (B) \( \frac{90 - 40\sqrt{3}}{11} \) (C) 2 (D) \( \frac{144 - 25\sqrt{3}}{44} \) (E) \( \frac{5}{2} \)

第20题解答

解答步骤

1. 理解题意

已知直圆锥底面半径 \( R = 5 \) ,高 \( H = 12 \) ,内部放置三个半径为 \( r \) 的全等球体。每个球体与其他两个球体相切,并且与圆锥的底面和侧面相切。要求求出 \( r \) 的值。


2. 建立几何模型

设圆锥顶点为 \( V \) ,底面圆心为 \( C \) 。三个球的球心位于同一水平面(与底面平行),且构成边长为 \( 2r \) 的等边三角形。 取其中一个球的球心为 \( A \) ,该球与底面切点为 \( B \) ,与侧面切点对应的母线为 \( \overline{VD} \) ,其中 \( D \) 在底面圆周上,且 \( B \) 在 \( \overline{CD} \) 上。 设 \( E \) 为 \( \overline{CV} \) 上一点,使得 \( \overline{ED} \) 经过球心 \( A \) 。


3. 球心水平位置

三个球心构成的等边三角形边长为 \( 2r \) ,其外心(也是重心)在圆锥的对称轴 \( CV \) 上。 球心 \( A \) 到轴 \( CV \) 的水平距离为等边三角形中心到顶点的距离: 等边三角形高为 \( 2r \cdot \frac{\sqrt{3}}{2} = r\sqrt{3} \) ,重心到顶点的距离为高的 \( \frac{2}{3} \) ,即 \[ BC = \frac{2}{3} \cdot r\sqrt{3} = \frac{2\sqrt{3}}{3} r \] 这里 \( BC \) 是 \( B \) 到 \( C \) 的水平距离,也是球心 \( A \) 在水平面上的投影到 \( C \) 的距离。


4. 圆锥母线与侧面切点关系

圆锥母线 \( VD \) 长度为: \[ VD = \sqrt{VC^2 + CD^2} = \sqrt{12^2 + 5^2} = \sqrt{144 + 25} = \sqrt{169} = 13 \] 在三角形 \( CDV \) 中, \( \overline{DE} \) 是 \( \angle CDV \) 的角平分线(因为球与侧面相切时,球心到侧面的垂线在母线上,且 \( A \) 在 \( DE \) 上, \( EA \perp VD \) )。 由角平分线定理: \[ \frac{CD}{CE} = \frac{DV}{EV} \] 代入 \( CD = 5 \) , \( DV = 13 \) , \( EV = CV - CE = 12 - CE \) : \[ \frac{5}{CE} = \frac{13}{12 - CE} \] 交叉相乘: \[ 5(12 - CE) = 13 \cdot CE \] \[ 60 - 5CE = 13CE \] \[ 60 = 18CE \] \[ CE = \frac{60}{18} = \frac{10}{3} \]


5. 利用相似三角形求 \( BD \)

在直角三角形 \( DAB \) 与 \( DEC \) 中: 更准确地说: \( \triangle DAB \) 与 \( \triangle DEC \) 相似,因为: - \( \angle DBA = \angle DCE = 90^\circ \) ( \( B \) 在底面, \( C \) 是圆心) - \( \angle BDA = \angle EDC \) (公共角) 所以: \[ \frac{BD}{CD} = \frac{AB}{CE} \] 即: \[ BD = AB \cdot \frac{CD}{CE} = r \cdot \frac{5}{\frac{10}{3}} = r \cdot \frac{5 \cdot 3}{10} = r \cdot \frac{15}{10} = \frac{3}{2} r \]


6. 建立 \( r \) 的方程

点 \( B \) 在底面半径 \( CD \) 上,且 \( BC + BD = CD = 5 \) : \[ BC + BD = \frac{2\sqrt{3}}{3} r + \frac{3}{2} r = 5 \] 通分(分母 6): \[ \frac{4\sqrt{3} r + 9 r}{6} = 5 \] \[ (4\sqrt{3} + 9) r = 30 \] \[ r = \frac{30}{9 + 4\sqrt{3}} \] 有理化分母: \[ r = \frac{30(9 - 4\sqrt{3})}{(9 + 4\sqrt{3})(9 - 4\sqrt{3})} \] 分母: \[ 81 - (4\sqrt{3})^2 = 81 - 48 = 33 \] 分子: \[ 30(9 - 4\sqrt{3}) = 270 - 120\sqrt{3} \] 所以: \[ r = \frac{270 - 120\sqrt{3}}{33} \] 分子分母同除以 3: \[ r = \frac{90 - 40\sqrt{3}}{11} \]


7. 最终答案

\[ \boxed{\frac{90 - 40\sqrt{3}}{11}} \] 对应选项 (B)。